LSAT and Law School Admissions Forum

Get expert LSAT preparation and law school admissions advice from PowerScore Test Preparation.

 Administrator
PowerScore Staff
  • PowerScore Staff
  • Posts: 8917
  • Joined: Feb 02, 2011
|
#26526
Complete Question Explanation

Weaken—CE, #%. The correct answer choice is (D)

The key to this author’s argument is the belief that Albritten’s water will contain at least 250 milligrams of salt per liter within the next few decades, making it unpalatable. This conclusion is based on the fact that Albritten’s water currently contains 100 milligrams of salt, significantly more than that of a nearby area. Of course, the belief that the salt content will increase by at least 150 mg stems from the assumption that before the road salting began it must have been much lower than the current 100 mg (that is, the author obviously assumes that it has already risen substantially in order to conclude that it will continue to rise substantially in the future). To weaken this argument, the correct answer choice should attack this assumption and show that the salting has not had much (or any) effect on the present salt levels. That would make it unlikely that the content would rise 150 mg in the next few decades.

Answer choice (A): The argument is about palatability (taste), not safety, so this answer choice is incorrect.

Answer choice (B): This answer choice strongly supports the conclusion, as the water would certainly be unpalatable if it reaches 400 mg/liter (since 250 is said to be the threshold).

Answer choice (C): The motive for salting the roads is not important to the argument; the correct answer must attack the water’s salt content and the presumed increase.

Answer choice (D): This is the correct answer choice. If the salt content has only risen 10 mg in the past 20 years (from 90 to 100), then it is quite unlikely that it will rise 150 in the next 20 years.

Answer choice (E): Since the argument concludes that the salt content will reach 250 if salting continues “at the present rate,” this answer choice does not address the argument.
 ellenb
  • Posts: 260
  • Joined: Oct 22, 2012
|
#12283
Dear Powerscore,

I had a quick question. I have read the explanations, however, I am still a bit confused as to why E is wrong? Is it because we do not know for sure that it will decrease, since in the stimulus it says that it will increase.

Thanks in advance!
User avatar
 Dave Killoran
PowerScore Staff
  • PowerScore Staff
  • Posts: 5852
  • Joined: Mar 25, 2011
|
#12289
Hi Ellen,

It is because the conclusion specifically states that "continuing the salting of Albritten’s roads at its present rate..." (italics added). Since the conclusion includes the idea that the rate stays constant, saying it will decrease doesn't hurt the argument.

For example, if you conclude that, "If I keep spending money at this rate I will go bankrupt soon," and someone replies, "You won't keep spending money at this rate," they haven't actually weakened your statement; they've just said it isn't likely to occur. The idea that "If I keep spending money at this rate I will go bankrupt soon" is still true.

Does that make sense? Please let me know.

Thanks!
 kells__w
  • Posts: 10
  • Joined: Mar 29, 2021
|
#86248
Hi,
I am having trouble understanding why E is wrong. If we are supposed to take the answers in a weaken question as true wouldn't E being true (stating the salting of the roads is likely to decrease as a fact) undermine the original argument in the stim that says that continuing the salting of the roads at its present rate will render the groundwater unpalatable weaker as it suggests the facts/ hypothetical situation presented by the author are incorrect?
Thanks in advance!
User avatar
 KelseyWoods
PowerScore Staff
  • PowerScore Staff
  • Posts: 1079
  • Joined: Jun 26, 2013
|
#86300
Hi Kells!

Whether the hypothetical situation the author gives us in the conclusion is actually going to happen or not is irrelevant to whether the predicted outcome will occur. The author is only making a prediction about what will happen if Albritten continues salting at its present rate. But the author is not saying that Albritten actually will continue salting at its present rate.

Let's apply this to a similar situation. Let's say your friend Nick, who is lactose-intolerant says: "If I eat that extra cheese pizza, my stomach is going to be a mess!" And then let's say Nick decides not to eat that pizza. Does that attack Nick's initial statement? Does Nick's decision not to eat the pizza have any effect on his original statement that if he did eat the pizza, it would upset his stomach? Nope! Nick is just predicting what would happen in a certain situation, but he's not saying that the situation is necessarily going to happen.

It's the same idea here. The author is just telling us what would happen if they continue salting at the present rate. The author is not telling us that Albritten will continue salting at the present rate. The author is making a prediction (saltwater will become palatable within the next few decades) based on a hypothetical situation (continuing salting at the present rate) that may or may not happen.

Incidentally, a hypothetical situation NOT happening really doesn't tell us anything about whether the initial prediction was true or not and, therefore, cannot be used to weaken such an argument. We'll never know whether Nick's stomach really would have been upset if he had eaten that cheese pizza. If Albritten decreases their salting, we'll never find out for sure what would have happened if they had continued at their present rate.

We need to weaken the prediction the author is making, not the hypothetical situation. We need an answer choice that suggests that even if the hypothetical situation occurs and Albritten does continue salting at the present rate, the predicted outcome (unpalatable groundwater) will not necessarily occur.

Hope this helps!

Best,
Kelsey
 lucano284@gmail.com
  • Posts: 1
  • Joined: Mar 13, 2024
|
#105819
I am having a hard time understanding why it is D the answer. I thought if the salt per liter goes up or is higher then 10 milligrams would not support the authors argument that the ground water would not be plantable.
User avatar
 Dana D
PowerScore Staff
  • PowerScore Staff
  • Posts: 117
  • Joined: Feb 06, 2024
|
#105830
Hey Lucano,

Focusing in on the conclusion of this argument - the author says continued salting of Albritten's roads at this rate will render the water unpalatable in the next few decades. To support this, the author compares the salt levels of Albritten to that of a nearby area's groundwater, which has significantly less salt per liter. The author implies that the heavy salting which started 20 years ago is the cause of the very salty groundwater Albritten now has, which is why if we continue at this rate the water will be unpalatable in a few decades.

In order to weaken a cause and effect argument like this, we have a couple of strategies - we can prove the effect happened before the cause, the cause happened without the effect, the effect happened without the cause, etc. We want an answer choice that employs one of these strategies.

Looking at answer choice (D), if the groundwater in Albritten was already pretty salty at 90 mg of salt per liter 20 years ago and now, 20 years later, it's at 100 mg per liter, then the sudden implementation of heavy salting of the roads didn't actually have as big of an effect on the salt in the groundwater as we initially thought. It seems as though the effect (very salty groundwater) was already present in Albritten even before the alleged 'cause' (heavy salting of the roads) took place. In this way, Answer Choice (D) weakens the argument.

Get the most out of your LSAT Prep Plus subscription.

Analyze and track your performance with our Testing and Analytics Package.